Quadrato e punti (sns 1998)

Rette, triangoli, cerchi, poliedri, ...
Rispondi
mark86
Messaggi: 260
Iscritto il: 01 gen 1970, 01:00

Quadrato e punti (sns 1998)

Messaggio da mark86 »

Dato un quadrato $ Q $ di lato unitario siano $ P_1 $, $ P_2 $, $ P_3 $, $ P_4 $, $ P_5 $ dei punti interni a Q. Sia $ d_{ij} $ la distanza fra $ P_i $ e $ P_j $.

a) Si dimostri che almeno una delle distanze $ d_{ij} $ è minore di $ \frac{\sqrt{2}}{2} $
b) Si può sostituire $ \frac{\sqrt{2}}{2} $ con un numero più piccolo?
Avatar utente
Boll
Messaggi: 1076
Iscritto il: 01 gen 1970, 01:00
Località: Piacenza

Messaggio da Boll »

Dividiamo il quadrato in 4 quadratini di area uguale (1/4) tracciando le congiungenti i punti medi parallele ai lati. Per il lemma dei cassetti avremo che due punti stanno sullo stesso quadratino. E' quindi evidente (se si vuole essere rigorosi sì può osservare che i due lati del triangolino rettangolo che si forma sono entrambi <1/2 e concludere con Pitagora) che la congiungente i due punti che stanno nello stesso rettangolo è minore della diagonale del quadrato stesso, che misura appunto $ \dfrac{\sqrt2}{2} $
mark86
Messaggi: 260
Iscritto il: 01 gen 1970, 01:00

Messaggio da mark86 »

proprio quello che avevo pensato io... quindi la risposta al secondo punto è NO! Giusto?
EvaristeG
Site Admin
Messaggi: 4896
Iscritto il: 01 gen 1970, 01:00
Località: Roma
Contatta:

Messaggio da EvaristeG »

Per quanto riguarda il secondo punto, se vuoi rispondere no, basta che tu esibisca una configuarzione di 5 punti in cui la minima distanza tra due di essi è proprio quella data.
In generale, in questo tipo di problemi, per dimostrare che un certo valore è il minimo o il massimo, basta dimostrare che è minorante o maggiorante e poi esibire un caso in cui esso viene effettivamente raggiunto.
mark86
Messaggi: 260
Iscritto il: 01 gen 1970, 01:00

Messaggio da mark86 »

Direi.... quattro punti coincidenti con i vertici e il quinto punto nell'intersezione con le diagonali, in questo caso la distanza minima è $ \frac{\sqrt{2}}{2} $... quale argomentazione si dorebbe usare per dire che questa configurazione mostra che non si può prendere un valore minore di $ \frac{\sqrt{2}}{2} $???
EvaristeG
Site Admin
Messaggi: 4896
Iscritto il: 01 gen 1970, 01:00
Località: Roma
Contatta:

Messaggio da EvaristeG »

Hmm... tu hai dimostrato (no, beh, l'ha fatto Boll, ma è lo stesso) che, comunque dati 5 punti nel quadrato, ve ne sono due la cui distanza è minore o uguale a L.
Con L=$ 1/\srtq{2} $

Vuoi sapere se è possibile che la stessa frase sia vera con L minore di quel valore ... cioè vuoi poter dire
comunque dati 5 punti nel quadrato, ve ne sono due la cui distanza è minore o uguale a L'
con L' minore strettamente di $ 1/\sqrt{2} $.

Bene, se tu mostri che esiste una configurazione di punti in cui la distanza minima è proprio quella, basta che verifichi questa proposizione 2) su quella configurazione ... essa sarà falsa, perchè la distanza minima tra due punti sarà $ 1/\sqrt{2}>L' $ e quindi non è vero che esistono sempre due punti a distanza minore o uguale di L', se questo L' è minore del nostro valore.
Quindi 1/sqrt{2} è il valore più piccolo che può avere L affinchè la affermazione sia vera...se infatti lo poni minore, i tuoi cinque punti fanno da controesempio.

Diciamolo algebricamente ... hai una espressione F(x); riesci a dimostrare che $ F(x)\leq A $ con A un numero reale. Se trovi $ x_0 $ tale che $ F(x_0)=A $ allora potrai affermare che A è il massimo di F; altrimenti potrai solo dire che la maggiore.
Ad esempio
$ f(x)=\frac{x^2+2}{x^2+1}\leq 100 $ è vero, ma 100 non è il massimo della funzione, infatti non esiste alcun x per cui f(x)=100; anzi, si ha $ f(x)\leq 2 $ e f(0)=2, quindi 2 è il massimo valore della nostra espressione.

Nel tuo caso, f associa ad una configurazione di 5 punti nel quadrato la distanza minima tra due di essi ... è una funzione ben definita e noi ne cerchiamo il massimo facendola spaziare su tutte le possibili configurazioni di 5 punti nel quadrato.
Riusciamo a dimostrare che questa funzione è sempre minore di $ 1/\sqrt{2} $. Se dimostriamo che in un certo punto assume proprio questo valore, sappiamo che non c'è una costante migliore che ne limita i possibili valori.

chiaro?
Rispondi